Diễn Đàn MathScope

Diễn Đàn MathScope (http://forum.mathscope.org/index.php)
-   Chuyên Đề (http://forum.mathscope.org/forumdisplay.php?f=60)
-   -   Khi nào một đa thức là bất khả quy? (http://forum.mathscope.org/showthread.php?t=1515)

n.t.tuan 03-01-2008 06:55 PM

Khi nào một đa thức là bất khả quy?
 
Topic này sẽ giới thiệu vài tiêu chuẩn bất khả quy của các đa thức , tôi không đăng các chứng minh kèm theo vì hoặc là các bạn đều biết rồi hoặc kiến thức của các bạn học sinh phổ thông không thể hiểu các chứng minh đó. Nếu có ai đó yêu cầu một chứng minh , tôi sẽ đăng nó nếu tôi biết.

k ở đây là Z,Q,... (nhìn như một vành)
Định nghĩa. Một đa thức f với hệ số trong k được gọi là khả quy trên k nếu f=gh, ở đây g và h là các đa thức bậc dương và có hệ số thuộc k. Trong các trường hợp khác f được gọi là bất khả quy trên k.

Giờ tôi đi vào giới thiệu luôn các tiêu chuẩn.

Định lý 1. (Eisenstein)
$f(x)=a_0+a_1x+...+a_nx^n\in\mathbb{Z}[x] $. Nếu có số nguyên tố p sao cho $a_n $ không chia hết cho p , $a_0,...,a_{n-1} $ chia hết cho p, và $a_0 $ không chia hết cho$ p^2 $ thì f là bất khả quy trên Z.

psquang_pbc 03-01-2008 07:25 PM

Chứng minh của Định lý 1 :

Giả sử lại rằng :

$P(x)=Q(x).R(x).(*) $

ở đây $Q(x) $ và $R(x) $ là các đa thức hệ số nguyên và :

$Q(x)=\sum_{i=0}^hb_ix^i $

$R(x)=\sum_{i=0}^kc_ix^i $

với $b_i\;(i=\overline{0,h})\;,c_j\;(j=\overline{0,k})\ $ là các số nguyên, $h,k>0,h+k=n $

Bằng cách đồng nhất hệ số của 2 vế đẳng thức $(*) $ ta có $a_0=b_h.c_k $. Do $a_0\;\vdots\;p $ nhưng $a\;\not\vdots\;p^2 $ nên hoặc $b_h\;\vdots \;p $ hoặc $c_k\;\vdots \;p. $

Giả sử $b_h\;\vdots \;p $.Ta có chuỗi các đẳng thức sau:

$a_{1}=b_hc_{k-1}+b_{h-1}c_k $

$a_{2}=b_hc_{k-2}+b_{h-1}c_{k-1}+b_{h-2}c_k $

$a_{h}=a_{n-k}=b_{h}.c_{k-h}+b_{h-1}.c_{k-h+1}+...+b_1.c_{k-1}+b_0.c_k $

Trong đẳng thức thứ nhất ta suy ra $b_{h-1}\;\vdots\;p $, tương tự ta cũng có $b_{h-2}\;\vdots\;p $ từ đẳng thức thứ 2. Quá trình đó tiếp tục đến lúc ta có $b_0\;\vdots\;p $ hay $a_n\;\vdots\;p $ vô lí.

Kết thúc chứng minh.

n.t.tuan 03-01-2008 09:47 PM

Định lý 2.
Cho p là một số nguyên tố. Nếu đa thức $f(x)=a_nx^n+...+a_0\in\mathbb{Z}[x] $ khả quy trên Z, và p chia hết các hệ số $a_0,...,a_{n-2} $ nhưng không chia hết $a_n $, và $p^2 $ không chia hết $a_0 $, thì p không chia hết $a_{n-1} $ và f phải có ít nhất một nghiệm hữu tỷ.

psquang_pbc 04-01-2008 06:24 AM

Định lý $2 $ là hệ quả của định lý sau :

Định lý $2' $

Cho $P(x)=\sum_{i=0}^n a_{n-i}i.x^i $ là đa thức hệ số nguyên và $k $ là $1 $ số tự nhiên sao cho $0\le k\le n-1 $. Giả sử tồn tại số nguyên tố $p $ sao cho:

$1, $ $a_0 $ không chia hết cho $p $.

$2, $ Những hệ số $a_{k+1},a_{k+2},...,a_n $ chia hết cho $p $.

$3, $ $a_n $ không chia hết cho $p^2 $.

Chứng minh khi đó P(x) có ước không phân tích được là G(x) mà bậc của đa thức này lớn hơn hoặc bằng $n-k. $

Với $k=0 $ ta nhận được tiêu chuẩn Eisenstein

Với $k=1 $ ta có định lý 2 của anh Tuân.

Với $k=n-2 $ thì tồn tại 1 nghiệm của P(x) không là số vô tỉ .

T.Courtin 04-01-2008 11:09 AM

Định lý 3.(Tiêu chuẩn Polya)
Cho $f\in\mathbb{Z}[x] $ bậc $n $ . Đặt $m=\left[\frac{n+1}{2}\right] $ .Giả sử $n $ số nguyên khác nhau $d_1,d_2,\ldots,d_n $ không là nghiệm của $f $ và thỏa mãn $f(d_i)<\frac{m!}{2^n} $ .
Khi đó $f $ bất khả quy.

psquang_pbc 04-01-2008 12:35 PM

Định lý 4.(Tiêu chuẩn Oskar Perron)

Cho đa thức $P(x) $ hệ số nguyên. Giả sử tồn tại số nguyên $b $ và số nguyên tố $p $ sao cho chúng thoả mãn điều kiện sau :

$1, P(b)=p $

$2, P(b-1)\not=0. $

3, Tất cả các nghiệm $\xi_i;\; ( i=\overline{1,n} ) $ của đa thức $P(x) $ thỏa mãn bất đẳng thức $Re\xi_i <b-\frac{1}{2} $. Khi đó đa thức $P(x) $ bất khả quy.

n.t.tuan 04-01-2008 06:13 PM

Trích:

Nguyên văn bởi n.t.tuan (Post 6621)
Định lý 2.
Cho p là một số nguyên tố. Nếu đa thức $f(x)=a_nx^n+...+a_0\in\mathbb{Z}[x] $ khả quy trên Z, và p chia hết các hệ số $a_0,...,a_{n-2} $ nhưng không chia hết $a_n $, và $p^2 $ không chia hết $a_0 $, thì p không chia hết $a_{n-1} $ và f phải có ít nhất một nghiệm hữu tỷ.

Dùng định lý này có thể giải được các bài 10,11 ở đây //http://ant.edu.ms/forum/showthread.php?t=1490 . Chú Quang post chứng minh của định lý 2 đi.

psquang_pbc 04-01-2008 06:52 PM

Để thư thả rồi em post chứng minh Định lý 2' luôn. Giờ mệt quá chả làm được gì, lên mạng spam tí thôi

n.t.tuan 04-01-2008 07:03 PM

Tùy chú, anh post tiêu chuẩn khác nhé!

Định lý 5.
Cho $f(x)=x^n+a_1x^{n-1}+...+a_n\in\mathbb{Z}[x],a_n\not = 0 $.
a)Nếu $|a_1|>1+|a_2|+...+|a_n| $ thì f là bất khả quy trên Z.
b)Nếu $|a_1|\geq1+|a_2|+...+|a_n| $ và $f(\pm 1)\not = 0 $ thì f là bất khả quy trên Z.

psquang_pbc 04-01-2008 07:10 PM

Tiếp theo là định lý 6 nhé :

Định lý 6 :

Cho $b\ge 3 $ là 1 số nguyên dương và $p $ là 1 số nguyên tố . Viết $p $ dưới dạng cơ số $b $ như sau :

$p=\sum_{i=0}^na_ib^{n-i} $

ở đây n là số tự nhiên $a_0\not=0 $ và $0\le a_i<b\; ( i=\overline{1,n} \; ) $ . Khi đó đa thức:

$P(x)=\sum_{i=0}^na_ix^{n-i} $

là đa thức bất khả quy.

Lam_sptn 04-01-2008 07:18 PM

Trích:

Nguyên văn bởi psquang_pbc (Post 6652)
Định lý $2 $ là hệ quả của định lý sau :

Định lý $2' $

Cho $P(x)=\sum_{i=0}^n a_{n-i}i.x^i $ là đa thức hệ số nguyên và $k $ là $1 $ số tự nhiên sao cho $0\le k\le n-1 $. Giả sử tồn tại số nguyên tố $p $ sao cho:

$1, $ $a_0 $ không chia hết cho $p $.

$2, $ Những hệ số $a_{k+1},a_{k+2},...,a_n $ chia hết cho $p $.

$3, $ $a_n $ không chia hết cho $p^2 $.

Chứng minh khi đó P(x) có ước không phân tích được là G(x) mà bậc của đa thức này lớn hơn hoặc bằng $n-k. $

Với $k=0 $ ta nhận được tiêu chuẩn Eisenstein

Với $k=1 $ ta có định lý 2 của anh Tuân.

Với $k=n-2 $ thì tồn tại 1 nghiệm của P(x) không là số vô tỉ .

Ai có thể post giúp chứng minh của 2' được không?

n.t.tuan 04-01-2008 07:42 PM

Trích:

Nguyên văn bởi n.t.tuan (Post 6712)

Định lý 5.
Cho $f(x)=x^n+a_1x^{n-1}+...+a_n\in\mathbb{Z}[x],a_n\not = 0 $.
a)Nếu $|a_1|>1+|a_2|+...+|a_n| $ thì f là bất khả quy trên Z.
b)Nếu $|a_1|\geq1+|a_2|+...+|a_n| $ và $f(\pm 1)\not = 0 $ thì f là bất khả quy trên Z.

Dùng Định lý này có thể giải được bài 14 ở topic ''Bài tập về đa thức bất khả quy'' trong box này.

zinxinh 13-03-2009 05:12 PM

Xét trên trường $Z_{p} $

namdung 14-03-2009 10:51 AM

Đề chọn đội tuyển của Trung Quốc vừa rồi có bài: Cho m > n > 1 là các số nguyên lẻ, chứng minh rằng đa thức $x^m + x^n + x + 1 $ bất khả quy.
==============
Trích:

Nguyên văn bởi Lam_sptn (Post 6719)
Ai có thể post giúp chứng minh của 2' được không?

Chứng minh của 2' hoàn toàn giống như chứng minh của tiêu chuẩn Eisentein thôi. Bạn cứ theo dõi kỹ là chứng minh được.

pte.alpha 13-04-2009 11:07 AM

Tôi mới đọc trên Crux có định lý khá hay sau về điều kiện khả quy của đa thức $F(x^2) $.

Định lý: Đa thức $F(x^2) $ là khả quy trên $Z[x] $ khi và chỉ khi hoặc $F(x) $ khả quy, hoặc $aF(x) = G^2(x) - xH^2(x) $ với $G(x), H(x) $ thuộc $Z[x] $, trong đó a = 1 hoặc -1.

Định lý trên được phát biểu cho vành cơ sở K bất kỳ nhưng tôi phát biểu cho Z cho nó gọn. Trong trường hợp tổng quát, a được thay bằng 1 phần tử đơn vị của K.

Ứng dụng. Chứng minh rằng nếu f(x) thuộc Z[x] là đa thức bất khả quy và |f(0)| không chính phương thì f(x^2) bất khả quy.


Múi giờ GMT. Hiện tại là 12:07 PM.

Powered by: vBulletin Copyright ©2000-2024, Jelsoft Enterprises Ltd.

[page compression: 18.84 k/20.08 k (6.16%)]